LSAT and Law School Admissions Forum

Get expert LSAT preparation and law school admissions advice from PowerScore Test Preparation.

 Administrator
PowerScore Staff
  • PowerScore Staff
  • Posts: 8929
  • Joined: Feb 02, 2011
|
#100990
Complete Question Explanation

Resolve the Paradox, #%. The correct answer choice is (C).

Answer choice (A):

Answer choice (B):

Answer choice (C): This is the correct answer choice.

Answer choice (D):

Answer choice (E):

This explanation is still in progress. Please post any questions below!
 alphonse92m@gmail.com
  • Posts: 5
  • Joined: Feb 14, 2021
|
#106531
Can someone explain why B isn't correct? The way I see it, if ticket prices are down, then more people will frequent the theaters and there will be more receipts. On the other hand, lower prices can't sustain the cost of operating a theater, which is why many theaters are closing.
User avatar
 Dana D
PowerScore Staff
  • PowerScore Staff
  • Posts: 276
  • Joined: Feb 06, 2024
|
#106568
Hey Alphonse,

Remember that on the LSAT, "some" can mean just 1 - there's no way for us to know in this case how many theaters lowered their prices. Therefore, we can't say that answer choice (B) most helps resolve the discrepancy.

Hope that helps!

Get the most out of your LSAT Prep Plus subscription.

Analyze and track your performance with our Testing and Analytics Package.